Mathematical Induction (The inductive step)

Click For Summary
To prove that for all n≥1, 10n - 1 is divisible by 9, the inductive step requires assuming that 10k - 1 is divisible by 9 and demonstrating that 10k + 1 - 1 is also divisible by 9. The key is to express 10k + 1 - 1 in terms of 10k - 1. By rewriting it as 10(10k - 1) + 9, it becomes clear that both terms are divisible by 9. This approach clarifies the relationship between the two expressions and allows for the completion of the proof. Understanding this algebraic manipulation is essential for successfully applying mathematical induction in this case.
pavel329
Messages
6
Reaction score
0
1. I don't understand how to prove this.
for all n≥1, 10n - 1 is divisible by 9.





3. I've done the basis step.
Now I'm on the inductive step.
I'm using (10k+1-1)/9=1.
I don't know where to go from there.
Using algebra just gets me down to 10k+1= 10. And I really don't think that's the answer.
All examples I've seen show me things like 1...2..3..n+1= n(n+1) or something along the lines. They already give me the equation. This one does not.
 
Last edited by a moderator:
Physics news on Phys.org
pavel329 said:
1. I don't understand how to prove this.
for all n≥1, 10n - 1 is divisible by 9.





3. I've done the basis step.
Now I'm on the inductive step.
I'm using (10k+1-1)/9=1.
This isn't right. You need to show that 10k+1-1 is divisible by 9, not that it is equal to 9. There is a difference. For example, 27 is divisible by 9, but the two numbers aren't equal.
pavel329 said:
I don't know where to go from there.
Using algebra just gets me down to 10k+1= 10. And I really don't think that's the answer.
All examples I've seen show me things like 1...2..3..n+1= n(n+1) or something along the lines. They already give me the equation. This one does not.

What do you have for your induction hypothesis?
 
Assume that 10k-1 is divisable by 9. Prove that it implies 10k+1-1 is also divisable by 9.

Try to bring 10k+1-1 to such form that it contain 10k-1.
10k+1=10*10k=> 10k+1-1=10*10k-1=10*10k-10+10-1=
10(10k-1)+9.
Can you proceed from here?


ehild
 
Question: A clock's minute hand has length 4 and its hour hand has length 3. What is the distance between the tips at the moment when it is increasing most rapidly?(Putnam Exam Question) Answer: Making assumption that both the hands moves at constant angular velocities, the answer is ## \sqrt{7} .## But don't you think this assumption is somewhat doubtful and wrong?

Similar threads

  • · Replies 2 ·
Replies
2
Views
2K
Replies
6
Views
2K
  • · Replies 4 ·
Replies
4
Views
2K
  • · Replies 6 ·
Replies
6
Views
2K
Replies
5
Views
2K
  • · Replies 3 ·
Replies
3
Views
2K
  • · Replies 3 ·
Replies
3
Views
2K
  • · Replies 5 ·
Replies
5
Views
2K
  • · Replies 23 ·
Replies
23
Views
2K
Replies
1
Views
2K